Which of the choices below could be a scientific problem?
a. Tomato plants require 8 hours of direct sunlight to thrive.
b. Plant a tomato seedling 6 inches deep a large pot.
C. If tomato plants receive 4 hours of direct sunlight, then they will thrive.
d. How much sunlight do tomato plants need to thrive?

Answers

Answer 1

Answer:

D. How much sunlight do tomato plants need to thrive?

Step-by-step explanation:


Related Questions

Solve 7x + 1 < 4(x - 2).

A. {x | x > 3}
B. {x | x > -3}
C. {x | x < -3}
D. {x | x < 3}

Answers

SoLuTiOn☞

Given:- 7x + 1 < 4(x - 2)

Solving It:-

7x + 1 < 4(x - 2)

7x + 1 < 4x - 8

7x - 4x < -8 -1

3x < -9

x < -9/3

x < -3

So Correct Solution Set Will Be

C. {x | x < -3}

Hope This Helps You
So your answer is C hope this help

Please help, explain too and I will give brainliest :)

Answers

Answer:

c

Step-by-step explanation:

Answer:

C. 37 degrees

Step-by-step explanation:

27, 36, 45 is just a larger 3, 4, 5 triangle. The smallest angle is across from the smallest side. With those in mind, we have 37, 53, and 90 degrees as the angles for a 3, 4, 5 triangle. We pair the 27, (the three when we reduce) with the smallest angle, the 37.

I'm sure you can do it with an actual formula, but i can't recall. Cheers

what would be the u to usub and what would be the steps to solving this integral?

Answers

Presumably, ln⁵(x) is the same as (ln(x))⁵ (as opposed to a quintuply-nested logarithm, log(log(log(log(log(x)))))).

Then substituting u = ln(x) and du = dx/x gives

[tex]\displaystyle\int\frac{\mathrm dx}{x\ln^5(x)} = \int\frac{\mathrm du}{u^5} = -\frac1{4u^4}+C = \boxed{-\frac1{4\ln^4(x)}+C}[/tex]

pls help me solve this multiplication fractions. (show work)​

Answers

Answer:

32:3/4

33:4/3

34:40

35:48

Answer:

32. 3/4
33. 4/3
34. 40
35. 384/7

Step-by-step explanation:

32. 9/10 • 5/6

Multiply 9 x 5/10 x 6

9 x 5/10 x 6 = 45/60

Then simplify 45/60 “by factoring out 45 and 60 and finding the Greatest Common Factor (GCF)”

45/60 = 3/4

•••••••••••••••••••••••••••••••••••••••••••••••••••

33. 16/21 • 7/4

Multiply 16 x 7/21 x 4

16 x 7/21 x 4 = 112/84

Then simplify 112/84 “by factoring out 112 and 84 and finding the Greatest Common Factor (GCF)”

112/84 = 4/3

•••••••••••••••••••••••••••••••••••••••••••••••••••

34. 8 4/7 • 4 4/6

Multiply 7 by 8 and add 4

7 x 8 + 4 = 60

Then add the denominator back

60/7

Multiply 6 x 4 and add 4

6 x 4 + 4 = 28

Then add the denominator back

28/6

We’re left with 60/7 • 28/6

Multiply 60 x 28/7 x 6

60 x 28/7 x 6 = 1680/42

Then simplify 1680/42 “by factoring out 1680 and 42 and finding the Greatest Common Factor (GCF)”

1680/42 = 40

••••••••••••••••••••••••••••••••••••••••••••••••

35. 7 4/8 • 6 2/5

Multiply 8 by 7 and add 4

8 x 7 + 4 = 60

Then add the denominator back

60/7

Multiply 5 x 6 and add 2

5 x 6 + 2 = 32

Then add the denominator back

32/5

We’re left with 60/7 and 32/5

Multiply 60 x 32/7 x 5

60 x 32/7 x 5 = 1920/35

Then simplify 1920/35 “by factoring out 1920 and 35 and finding the Greatest Common Factor (GCF)”

1920/35 = 384/7

••••••••••••••••••••••••••••••••••••••••••••••••

please mark me as brainliest! This took me effort

Does this graph show a function? Explain how you know.

Answers

Answer:

A is the correct one

cause according to function rule the vertical line should cut only on a one point to be function

as here we can see that vertical line cuts here at two point

The slope of a line is 5/9 and the slope of another line is -975. The two lines
are

Answers

Answer:

the third option - they are perpendicular to each other.

Step-by-step explanation:

for a perpendicular slope we need to exchange the x and y values (remember, a slope is the ratio of y/x) and flip the sign.

and that is exactly what happened here.

Which equation represents a line that passes through ( -2 , 4 ) and has the slope of 2/5

Answers

Answer:

y = 2/5x +24/5

Step-by-step explanation:

The slope intercept form of a line is

y = mx+b where m is the slope and b is the y intercept

y = 2/5 x +b

Substitute the point into the equation and solve for b

4 = 2/5(-2)+b

4 = -4/5 +b

Add 4/5 to each side

20/5 +4/5 = b

24/5 = b

y = 2/5x +24/5

Which table shows a proportional relationship between a and b?

Answers

Answer:

It is c

Step-by-step explanation:

No idea how to do

[tex]\frac{4r + 20}{r + 5}[/tex]

when r ≠ 5 ?

Answers

Factorize the numerator:

4r + 20 = 4r + 4×5 = 4 (r + 5)

I think you meant to say r ≠ -5, which means r + 5 ≠ 0, so that the denominator is never zero and so the expression is defined (no division by zero). This lets you cancel the factor of r + 5 in the numerator with the one in the denominator:

(4r + 20)/(r + 5) = 4 (r + 5)/(r + 5) = 4

Whats his mistake and the correct awnser

Answers

Answer:

θ ≈ 35.7°

Step-by-step explanation:

His mistake was in using the wrong trig. ratio

Instead of using cosine he should have used the sine ratio

sinθ = [tex]\frac{opposite}{hypotenuse}[/tex] = [tex]\frac{7}{12}[/tex] , then

θ = [tex]sin^{-1}[/tex] ([tex]\frac{7}{12}[/tex] ) ≈ 35.7° ( to the nearest tenth )


Type the correct answer in the box. Use numerals instead of words. If necessary, use / for the fraction bar.
If x+2 is a factor of x^3-6x^2-11x+k then k=

Answers

Answer:

k=10

Step-by-step explanation:

Brainliest please~

Answer:

K=10

Step-by-step explanation:

Correct on Plato

what is the solution to this equation?
5x-4+3x=36

A. x=16
B. x=5
C. x=20
D. x=4

Answers

B. x = 5

tip : if in a rush just plug in the number and see if its true

8x - 4 = 36

x = 5

i will brainliest
avoid spam

Express each of the following percentages as a fraction and simplify it.

(i) 25% (ii) 40% (iii) 16% (iv) 150%

(v) 120% (vi) 58% (vii) 32% (viii) 175%

Answers

[tex]\boxed{\large{\bold{\textbf{\textsf{{\color{blue}{Answer}}}}}}:)}[/tex]

[tex]\sf{25\%=\dfrac{25}{100}=\dfrac{1}{4} }[/tex]

[tex]\sf{ 40\%=\dfrac{40}{100}=\dfrac{2}{5} }[/tex]

[tex]\sf{16\%=\dfrac{16}{100}=\dfrac{4}{25} }[/tex]

[tex]\sf{150\%=\dfrac{150}{100}=\dfrac{3}{2} }[/tex]

[tex]\sf{120\%=\dfrac{120}{100}=\dfrac{6}{5} }[/tex]

[tex]\sf{58\%=\dfrac{58}{100}=\dfrac{29}{50} }[/tex]

[tex]\sf{32\%=\dfrac{32}{100}=\dfrac{8}{25} }[/tex]

[tex]\sf{175\%=\dfrac{175}{100}=\dfrac{7}{4} }[/tex]

[tex]\sf{ }[/tex]

[tex]\sf{ }[/tex]

[tex]\sf{ }[/tex]

[tex]\sf{ }[/tex]

[tex]\sf{ }[/tex]

[tex]\sf{ }[/tex]

[tex]\hookrightarrow\sf{ 25\% =\dfrac{25}{100}=\boxed{\bf \dfrac{1}{4} }} \\\hookrightarrow\sf{ 40\% =\dfrac{40}{100}=\boxed{\bf \dfrac{2}{5} }} \\\hookrightarrow \sf{16\%=\dfrac{16}{100}=\boxed{\bf\dfrac{4}{25}} } \\ \hookrightarrow\sf{150\%=\dfrac{150}{100}=\boxed{\bf\dfrac{3}{2} }} \\ \hookrightarrow\sf{120\%=\dfrac{120}{100}=\boxed{\bf\dfrac{6}{5} }} \\\hookrightarrow \sf{58\%=\dfrac{58}{100}=\boxed{\bf\dfrac{29}{50}} } \\\hookrightarrow \sf{32\%=\dfrac{32}{100}=\boxed{\bf\dfrac{8}{25} }} \\ \hookrightarrow\sf{175\%=\dfrac{175}{100}=\boxed{\bf\dfrac{7}{4}} } \\ [/tex]

Can someone please help, I'll give out 20 points !

Question - Where is the blue point on the number line?

Answers

I believe it would be negative three (-4) since the lines are thinner however; there is a chance they are skip counting by 2. In that case the answer would be negative six (-6).

Answer:

-6

Step-by-step explanation:

The number line is created by skip counting of 2.

So, -2 ,  then -4 and then -6.

So,  blue dotted point is (-6)

Lines 3x-2y+7=0 and 6x+ay-18=0 is perpendicular. What is the value of a?

1/9
9
-9
-1/9

Pls answer​

Answers

Answer:

[tex]\boxed{\sf a = 9 }[/tex]

Step-by-step explanation:

Two lines are given to us which are perpendicular to each other and we need to find out the value of a . The given equations are ,

[tex]\sf\longrightarrow 3x - 2y +7=0 [/tex]

[tex]\sf\longrightarrow 6x +ay -18 = 0 [/tex]

Step 1 : Convert the equations in slope intercept form of the line .

[tex]\sf\longrightarrow y = \dfrac{3x}{2} +\dfrac{ 7 }{2}[/tex]

and ,

[tex]\sf\longrightarrow y = -\dfrac{6x }{a}+\dfrac{18}{a} [/tex]

Step 2: Find the slope of the lines :-

Now we know that the product of slope of two perpendicular lines is -1. Therefore , from Slope Intercept Form of the line we can say that the slope of first line is ,

[tex]\sf\longrightarrow Slope_1 = \dfrac{3}{2} [/tex]

And the slope of the second line is ,

[tex]\sf\longrightarrow Slope_2 =\dfrac{-6}{a} [/tex]

Step 3: Multiply the slopes :-

[tex]\sf\longrightarrow \dfrac{3}{2}\times \dfrac{-6}{a}= -1 [/tex]

Multiply ,

[tex]\sf\longrightarrow \dfrac{-9}{a}= -1[/tex]

Multiply both sides by a ,

[tex]\sf\longrightarrow (-1)a = -9 [/tex]

Divide both sides by -1 ,

[tex]\sf\longrightarrow \boxed{\blue{\sf a = 9 }} [/tex]

Hence the value of a is 9 .

63.89 X n = .6389 Solve for n *
A. 10
B. 100
C. .01
D. .1

Answers

Answer:

c. .01

Step-by-step explanation:

.6389 divided by 63.89

(C)
63.89*n=.6389

.6389/63.89=n
0.01=n

Which of the following represents x = 1/3y written in general form?
3x - y = 0
X-1/3y=0
1/3x-y=0

Answers

Answer:

x-⅓y=0

Step-by-step explanation:

thats the answer

brainly please

Answer: 3x-y=0

Step-by-step explanation:

X = 1/3y

X × 3 = 1/3 y × 3

3x = y

3x - y = y- y

3x- y = 0

Graph the following equations by plotting 3 points and then connecting the points to form a line. Show your work to find the 3 points and then list the ordered pairs you used as part of your answer.

Answers

Answer:

The way I would tackle these types of problems would be to make a table, then graph the points using the results I get.

Step-by-step explanation:

So explaining the table is kinda hard since I suck at explaining, so I've attached a file showing the table I've made for number 1.

First Step:

Make the table. Make it a 3x4 table. For the top row, put y, the equation (I put 'y=2x' as shown in number 1), then x. Then for the left column, go down by one box, then put: -1, 0, 1. Really, you can put any numbers, but to make it easy for yourself, just write the numbers I've listed. These will be your y-values you'll be substituting.

Second Step:

a.) Plugging in the numbers. For the middle column, I've put down y=2x in each box, as shown in my table. Then for each of those boxes, plug in the numbers to the left of them and substitute them for the y-value. For example, look at the second row. The left box has '-1', so I replaced the 'y' in the equation and got '(-1)=2x'.

b.) After substituting the y-values, solve for x. For this case, you'll need to divide both sides by 2. If it doesn't make much sense, think about a seesaw. If you have 5 pounds on both sides, the seesaw will stay balanced. Add an additional 5 pounds to one side, then the seesaw will start tipping towards the heavier sides. So in order to keep an equation balanced, you must do any action to both sides of the equation. After diving both sides by 2, you'll get the value for x.

Final Step:

After finding the x-values for each of the y-values, it's time to graph the equations. Remember, x is left to right and y is down to up. To graph them, let's use the second row for example. y=-1 and x=-0.5. So all you need to do is to start from the origin (0,0), then go half a unit to the left, as x is negative, then go a unit down, as y is negative, then make a point. The rest should be pretty straight forward.

After making all three points, just get a ruler or any straight edge, then align it along those three points, then just make a line, and boom! You've solved number 1! For the purpose of saving my time, I'm pretty sure you can solve the rest using what I've just told you. Good luck!

Can someone help me with this math homework please!

Answers

Given:

The sequence is:

-3, 5, -7, 9, -11,...

To find:

The correct statement about the given sequence.

Solution:

We have,

-3, 5, -7, 9, -11,...

Here, "..." means there are more terms in the sequence. So, the sequence has more than 5 terms and option A is incorrect.

From the given sequence it is clear that the 4th term is 9. So, option B is correct.

Clearly, 5th terms is -11. So, [tex]f(5)=-11[/tex] and option C is incorrect.

The domain of a sequence is always the set of natural numbers. So, option D is correct.

In the given sequence the 4th term is 9. It means the point (4,9) lies on the graph of the sequence. So, option E is correct.

Therefore, the correct options are B, D, E.

Use elimination to solve the system 4x + 7y = –14 and 8x + 5y = 8 for x. Round to the nearest tenth.

Answers

Answer:

X= 7/2

Step-by-step explanation:

Multiply first equation by -2 and second equation by 1

-2(4x+7y=-14)

1(8x+5y= 8)

= -8x -14y =28

8x+5y=8

Add these equations to eliminate X:

-9y=36

-9y/-9 = 36/-9

y=- 4

Plug in Y to find X

4x + 7y = -14

4x+7(-4) = -14

4x-28+28= -14+28

4x= 14

X= 7/2

Answered by Gauthmath

Answer: x=3 and 1/2

Step-by-step explanation: Happy to Help (0.<)

Make d the subject h=d/3+2

Answers

Answer:

[tex]d = 3h - 6[/tex]

Step-by-step explanation:

Step 1:  Solve for d

I am assuming that you are asking for me to solve for the variable d.  Let me know if what I am doing is not what you want.

[tex]h = \frac{d}{3} + 2[/tex]

[tex]h - 2 = \frac{d}{3} + 2 - 2[/tex]

[tex](h - 2) * 3 = \frac{d}{3} * 3[/tex]

[tex]3h - 6 = d[/tex]

Answer:  [tex]d = 3h - 6[/tex]

A man earned $3000 the first year he worked. If he received a raise of $600 at the end of each year, what was his salary during the 15th year?

A) $8400
B) None of these
C) $11,400
D) $12,000

Answers

Answer:

$11,400

Step-by-step explanation:

3,000+14(600)=11,400

We know that it expands by $600 a year and we are looking for the salary during the 15th year.  During the 15th year, it could be using the salary of the 14th year since it is during not after.  We then add that to the base salary of $3,000 so then we have $11,400.

Rate of Change: Based on the equation determine the rate of change by looking at the coefficients.

Hint: Remember: y=mx+b (m=slope)

1. y=-x+ 3
2. y=3x-1
3. y= 8 + x2

Answers

Answer:

1) -1

2) 3

3) 2

combine like terms and simplify (−3 + 4x2 − 4x) + (−1 − 3x2− 4)

Answers

Answer:

x² - 4x - 8

General Formulas and Concepts:

Algebra I

Terms/Coefficients

Step-by-step explanation:

Step 1: Define

Identify

(-3 + 4x² - 4x) + (-1 - 3x² - 4)

Step 2: Simplify

Combine like terms (x²):                                                                                   x² - 3 - 4x - 1 - 4Combine like terms:                                                                                         x² - 4x - 8

A) If {3,1,-1,-3} is the range of the function f(x) =2x+3/5, find its domain.
B) Which element in the range has pre-image 8 under the function of f:x->3x+1/5? ​

Answers

Answer:

64

Step-by-step explanation:

1=879=96889=

Rose’s checking account has a balance of Negative 54 dollars. She makes no more withdrawals but deposits $15 per day in her account over the next five days. On the sixth day, she makes no deposits, but the bank charges a maintenance fee of $5. What is the balance in her account on the sixth day? $16 $26 $124 $134

Answers

Answer:

16

Step-by-step explanation:

Her account balance on the sixth day is sixteen. This is because -54 +75 is 21 then you subtract 5 and get 16. (15 times 5 is 75)  

Answer:

16

Step-by-step explanation:

Starting balance: -54

Deposits +15

                +15

                 +15

                 +15

                +15

for a  total of +75

-54+75 = 21

Maintenance fee: - 5

21 - 5 = 16

Determine which quadratic equation has two
non-real solutions.

Answers

Answer:

Hello,

answer C : -3x²-3x-2=0 (or 3x²+3x+2=0)

Step-by-step explanation:

A: delta=12²-4*2*18=0 not negative

B: delta= 5²+4*3*1=37 not negative

C: 3x²+2x+2=0, delta=2²-4*3*2=16-24=-8 is negative: roots are complex

D: delta 16²-4*2*2=240 not negative

Find the value of x.

Answers

Ok the answer is 7524482819484

What is the greatest common factor of the polynomial below?
12x2-9x
A. 3x2
B. 3x
C. 4x2
D. 4x

Answers

Answer:

3x

Step-by-step explanation:

factoring it we get

3x(4x-3)

Which graph represents an exponential function?

Answers

Answer: where's the pic?

Step-by-step explanation:

Other Questions
Evaluate x 2 5 y 3 when x = 4 and y =1 A ball is thrown from an initial height of 7 feet with an initial upward velocity of 23 ft/s. The ball's height h (in feet) after 1 seconds is given by the following.h = 7+23t-16t^2Find all values of 1 for which the ball's height is 15 feet. Solve the system of equations.6x = 3-2x + y = 141.What number would you multiply the second equationby in order to eliminate the x-terms when adding to thefirst equation?2.What number would you multiply the first equation by inorder to eliminate the y-terms when adding to thesecond equation? 9Drag each label to the correct location.Identify each item as a cause or an effect of imperialism in Southeast Asia,Locals suffered in poverty,Westen nations grew richer byExploidng colonized nations,Western nations wanted to profitfrom weaker, resource-rich nations,Internal conflicts arose becauseof diverse communities living ina single nation,Cause of ImperialismEffect of Imperialism The following information is available for the first month of operations of Bahadir Company, a manufacturer of mechanical pencils: Sales $278,670 Gross profit 162,460 Cost of goods manufactured 139,340 Indirect labor 60,470 Factory depreciation 9,200 Materials purchased 85,830 Total manufacturing costs for the period 160,240 Materials inventory, ending 11,430Using the above information, determine the following missing amounts a. Cost of goods sold b. Finished goods inventory at the end of the month c. Direct materials cost d. Direct labor cost e. Work in process inventory at the end of the month 1. 2. 3. 4. 5. Correct answer =thanksBest answer=brainliestRandom answers or unexplained answers=reportedBe careful while giving answers answer if you know otherwise don't Minstrel Manufacturing uses a job order costing system. During one month, Minstrel purchased $212,400 of raw materials on credit; issued materials to production of $207,000 of which $33,600 were indirect. Minstrel incurred a factory payroll of $157,200, of which $43,600 was indirect labor. Minstrel uses a predetermined overhead application rate of 150% of direct labor cost. Minstrel's beginning and ending Work in Process Inventory are $16,700 and $29,400 respectively. Compute the cost of jobs transferred to Finished Goods Inventory. Explain how you can find the constant of proportionality from a graph representing a proportional relationship when it shows a point with an x - value of 1 and if it doesn't show an x - value of 1 . Brewery Companys debt to Credit Service is past due. Credit obtains a judgment against Brewery, but the firm refuses to pay. Credit asks the court to order the seizure of Brewerys property. This is a request for What routine do the workers of the Party go through each day, which is a great example of herd-poisoning Which rule is a recursive rule for the sequence 1, -6, 36, -216,...?( I'll put the answers a screenshot )PLZZ Help I've been stuck for so long write a short note on geosynchronous orbit i. Your pronunciation is not so good. SOMEONE PLEASE HELP ME ASAP !!!Mrs. Pregnals's biology classes are going on a field trip. She has them break into groups of three or four students in order to do labs. She has a total of 120 students and 32 total groups are formed.If t represents the number of three person groups and frepresents the number of four person groups, then write a system of equations that models this situation.How many four person groups are there? Only an algebraic solution will receive full credit.PLEASE HELP !!!! A covalent bond is formed by the following process Does the government get its power from people, or do people get their power from the government pls help and pls provide workings Suppose f(x)=x^2. What is the graph of g(x)=1/2f(x)? A chemical reaction in a bomb calorimeter evolves 3.86 kJ of energy in the form of heat. If the temperature of the bomb calorimeter increases by 4.17 K, what is the heat capacity of the calorimeter? :902:What is the value of x?